Diễn Đàn MathScope

Diễn Đàn MathScope (http://forum.mathscope.org/index.php)
-   2011 (http://forum.mathscope.org/forumdisplay.php?f=176)
-   -   Việt Nam Team Selection Test 2011-Đề thi, Đáp án và Danh sách Đội tuyển (http://forum.mathscope.org/showthread.php?t=18448)

n.v.thanh 08-04-2011 05:56 PM

Việt Nam Team Selection Test 2011-Đề thi, Đáp án và Danh sách Đội tuyển
 
Topic về kì thi chọn đội tuyển Việt Nam tham dự kì thi IMO 2011

-----------------------------------------------------

Đến hẹn lại lên,sau kì thi VMO 2011 diễn ra cách đây gần 2 tháng thì vào ngày mai và ngày kia Bộ GDĐT sẽ tổ chức kì thi chọn đội tuyển VN tham dự kì thi Olympic [Only registered and activated users can see links. Click Here To Register...] diễn ra ở Hà Lan :D.
Nvthanh lại một lần nữa lập ra topic này nhằm tạo một nơi thảo luận có khoa học về kì thi này. Về cơ bản kì thi sẽ diễn ra vào hai ngày 8 và mùng 9 tháng 4 năm 2011(tức ngày mai và ngày kia) với sự góp mặt của 42 thí sinh đến từ khắp các vùng miền hải đảo của Tổ quốc Việt Nam yêu dấu.
Về công việc ngày mai thì do đội ngũ Mod của forum không ai tham gia kì thi quan trọng này nên vấn đề đề đóm có lẽ cứ mạnh ai nấy viết rồi các Mod sẽ edit thành một đề hoàn chỉnh vậy.

Vậy nhé, đã quán xuyến công việc xong. Cho Nvthanh gửi lời chúc tới các thí sinh, đặc biệt là các thí sinh đến từ Chuyên KHTN, cố giật vài vé đi Amsterdam nhé:). Mọi người có thể chúc tụng thoải mái nhưng sáng mai T sẽ del để sao cho đề bài sẽ là ở Post 2, mong mọi người thông cảm.


n.v.thanh 09-04-2011 12:00 PM

Đề thi chọn đội tuyển Việt Nam tham dự kì thi Olympic Toán quốc tế 2011


Ngày thi thứ nhất 9/04/2011
Thời gian làm bài 240 phút


Bài 1(5 điểm)
Tại điểm (1;1) của mặt phẳng tọa độ Oxy có một con cào cào.Từ điểm đó,con cào cào chỉ nhảy đến các điểm nguyên dương khác theo quy tắc: từ điểm nguyên dương A,con cào cào nhảy đến điểm nguyên dương B nếu tam giác AOB có diện tích bằng $\dfrac{1}{2} $.
1/Tìm tất cả các điểm nguyên dương (m;n) mà con cào cào có thể nhảy đến sau một số hữu hạn bước,xuất phát từ điểm (1;1).
2/Giả sử (m;n) là một điểm nguyên dương có tính chất đã nêu ở câu 1/.Chứng minh rằng tồn tại một cách nhảy của con cào cào từ điểm (1;1) đến điểm (m;n) mà số bước nhảy không vượt quá |m-n|.
(Điểm (x;y) gọi là điểm nguyên dương nếu x và y là các số nguyên dương).

Bài 2(7,0 điểm)
Trên mặt phẳng cho (O ) và một điểm A nằm ngoài đường tròn đó.Qua A kẻ các tiếp tuyến tới (O),gọi B,C là tiếp điểm.Xét một điểm Pdi động trên tia đối của tia BA,Q là điểm di động trên tia đối của tia CA sao cho đường thẳng PQ tiếp xúc với (O).Qua P kẻ đường thằng song song với AC,cắt BC tại E.Qua Q kẻ đường thẳng song song với AB cắt BC tại F.Chứng minh rằng
1/Đường thẳng EQ luôn đi qua một điểm cố định M và FP luôn đi qua một điểm cố định N.
2/Tích PM.QN không đổi.

Bài 3(8 điểm)
Cho số nguyên $n\geq 3 $.Xét $n $ số thực $x_1,x_2,\ldots,x_n $ thỏa mãn đồng thời các điều kiện sau:

i/ $x_1\geq x_2\geq x_2\geq \ldots \geq x_n $
ii/ $x_1+x_2+\ldots+x_n=0 $
iii/$x_1^2+x_2^2+\ldots+x_n^2=n(n-1) $

Tìm giá trị lớn nhất và giá trị nhỏ nhất của tổng $S=x_1+x_2 $

n.v.thanh 09-04-2011 12:18 PM

1 Attachment(s)
Ngày thi thứ hai 10/4/2011
Thời gian làm bài 240 phút
Bài 4 (6,0 điểm)
Cho dãy ${a_n} $ thỏa mãn $a_0=1,a_1=3 $ và $a_{n+2}=1+\left \lfloor \frac{a^2_{n+1}}{a_n} \right \rfloor $ với mọi $n\geq0 $
Chứng minh rằng
$a_n.a_{n+2}-a^2_{n+1}=2^n $ với mọi số tự nhiên $n $

Bài 5(7,0 điểm)

Tìm tất cả các số nguyên dương $n $ sao cho $A=2^{n+2}.(2^n-1)-8.3^n+1 $ là số chính phương.

Bài 6(7,0 điểm)
Cho n là một số nguyên lớn hơn 1.Có n học sinh ngồi quanh một chiếc bàn tròn,mỗi em có một số kẹo (có thể có em không có một chiếc kẹo nào) và tổng số kẹo của tất cả các em là một bội của n.Các em thực hiện việc chuyển kẹo như sau:
Với số kẹo mỗi em có lúc đầu,có ít nhất một em có nhiều kẹo hơn bạn ngồi bên phải mình thì một em (tùy ý) trong những em như thế chuyển một chiếc kẹo của mình cho bạn ngồi ngay bên phải.Với số kẹo mỗi em có sau lần chuyển thứ nhất,nếu có ít nhất một em có nhiều kẹo hơn bạn ngồi bên phải thì một em (tùy ý) trong số những em như thế lại tiếp tục chuyển 1 chiếc lẹo của mình cho bạn ngồi bên phải.Quá trình chuyển kẹo cứ thế được tiếp tục.
Chứng minh rằng sau một số hữu hạn lần chuyển kẹo như vậy,tất cả các em đều có số kẹo như nhau.
Hết

huynhcongbang 09-04-2011 03:45 PM

Mình ủng hộ bài 2 trước:
Trước hết, ta chứng minh bổ đề sau:
* Cho tam giác ABC ngoại tiếp (I) có tiếp điểm của (I) lên AB, AC lần lượt là E, F. Đường thẳng qua B, song song với AC cắt EF tại K; CK cắt AB tại G. Chứng minh rằng tam giác AGI vuông tại I.

Chứng minh:
Do BK // AC nên tam giác BKF cân tại B, suy ra: $BK=BF = p-b $.

[Only registered and activated users can see links. Click Here To Register...]

Theo định lí Thales thì:
$\frac{BG}{AG}=\frac{BK}{AC}=\frac{p-b}{b} \Rightarrow \frac{AB}{AG} = \frac{p}{b}\Rightarrow AG = \frac{bc}{p} $
Mà $AF=p-a $ nên $\frac{AF}{AG}=\frac{p(p-a)}{bc} $.
Ta cũng có: $AI = \frac{AF}{\sin \frac{A}{2}}, AH = AF. \sin \frac{A}{2} $.
Do đó: $\frac{AH}{AI}=\sin^2 \frac{A}{2} = \frac{1-\cos A}{2} = \frac{1-\dfrac{b^2+c^2-a^2}{2bc}}{2} = \frac{p(p-a)}{bc} $
Suy ra: $\frac{AF}{AG} =\frac{AH}{AI} $.
Tức là AGI vuông tại I.
Bổ đề được chứng minh.

Trở lại bài toán:

[Only registered and activated users can see links. Click Here To Register...]

1/ Gọi M, N lần lượt là giao điểm của QE với AB và PF với AC.
Theo bổ đề trên, ta thấy rằng tam giác OMA và ONA lần lượt vuông tại O nên các điểm M, N cố định.
2/ Đặt $AB=AC=a, BP=x, CQ=y $. Chu vi của tam giác APQ là $2(a+x+y) $.
Theo bổ đề trên, ta tính được:
$PM =AP - \frac{2AP.PQ}{AP+AQ+PQ}=\frac{(a+x)x}{a+x+y} $ và
$QN =AQ - \frac{2AQ.PQ}{AP+AQ+PQ}=\frac{(a+y)y}{a+x+y} $.
Ta sẽ chứng minh rằng $\frac{xy(a+x)(a+y)}{(a+x+y)^2} $ không đổi.
Thật vậy:
Diện tích của tam giác APQ cùng bằng:
$R(AP+AQ+PQ) = \sin \widehat{BAC}.AP.AQ \Leftrightarrow \frac{(a+x)(a+y)}{a+x+y}=\frac{R}{ \sin \widehat{BAC}} $.
Tức là tỉ số: $\frac{(a+x)(a+y)}{a+x+y} = k $ không đổi, với $k=\frac{R}{ \sin \widehat{BAC}} $
Từ $(a+x)(a+y)=k(a+x+y) \Leftrightarrow a(a+x+y)+xy = k(a+x+y) \Leftrightarrow a+ \frac{xy}{a+x+y} = k $, suy ra tỉ số $\frac{xy}{a+x+y} $ cũng không đổi.
Ta có đpcm.

shido_soichua 09-04-2011 06:28 PM

Thử bài 3 phát :D
Do $x_1^2+x_2^2+\ldots+x_n^2=n(n-1) $ nên $x_1^2+x_2^2=n(n-1)-x_2^2\ldots+x_n^2 $
Lại có
$(n-1)(x_3^2+x_4^2+.......+x_n^2)\geq (x_3+x_4+.....+x_n)^{2}= (x_1+x_2)^{2} $
$(x_1^2+x_2^2)\geq (x_1+x_2)^2 $
nên suy ra $n(x_1+x_2)^2\leq 2n(n-1)(n-2) $
Từ đấy suy ra $Max f=\sqrt{2(n-1)(n-2)} $ khi $x_1=x_2=\sqrt{\frac{(n-1)(n-2)}{2}} $ và $x_3=x_4=......x_n=-\sqrt{2(n-1} $
$min f=-\sqrt{2(n-1)(n-2)} $ khi $x_1=x_2=-\sqrt{\frac{(n-1)(n-2)}{2}} $ và $x_3=x_4=......x_n=\sqrt{2(n-1} $
Ôi min sai mất rùi

huynhcongbang 09-04-2011 10:01 PM

Trích:

Nguyên văn bởi nvthanh1994 (Post 89694)
Bài 1
Trên mặt phẳng tọa độ có một con cào cào ở điểm $(1;1) $.Nó có thể nhảy từ A sang B khi tam giác OAB có diện tích bằng $\dfrac{1}{2} $
1. Tìm các điểm $(m,n) $ sao cho con cào cào có thể nhảy đến đó sau hữu hạn bước
2.CMR con cào cào có thể nhảy đến $(m,n) $ kể trên sau ít hơn $|m-n| $ bước
[/TEX]

Bài này có lẽ liên quan đến một bài dãy số.
Gọi $A(x_1,y_1), B(x_2,y_2) $ là tọa độ của các điểm mà con cào cào có thể nhảy qua.
Diện tích tam giác OAB chính là:
$S_{OAB} = \frac{1}{2}|x_1y_2-x_2y_1| $.
Do đó, các điểm trong đề bài thuộc dãy số xác định như sau:
$x_0=y_0=1,x_{n+1}y_n-x_ny_{n+1} = \pm 1, n=0, 1,2,... $.
Đến đây đưa bài toán về tìm điều kiện của m và n sao cho $(m,n) $ thuộc dãy số trên. =p~

Traum 09-04-2011 10:08 PM

Bài 1:

Nhận xét 1:Giả sử tại một thời điểm, con cào cào ở đỉnh $A = (a,b) $ thì nó nhảy sang được đỉnh $B = (c,d) $ khi và chỉ khi $|ad-bc| = 1 $.

Thật vậy diện tích của tam giác $AOB $ là $\frac{1}{2}|ad-bc| $, nên con cào cào có thể nhảy từ đỉnh $A $ sang $B $ khi và chỉ khi $|ad-bc|=1 $. Nhận xét 1 được chứng minh.

Nhận xét 2: Với $(m,n) $ là cặp nguyên tố cùng nhau thì tồn tại a và b nguyên tố cùng nhau sao cho $|mb-na| = 1 $ và $|a-b|\le |m-n|-1 $

Chứng minh: không mất tổng quát, giả sử $m>n $, hiển nhiên tồn tại $1\le b\le n-1 $ sao cho $mb-1 $ chia hết cho $ n $
đặt $ a = (mb-1)/n $ ta có $ a<m $ và $ |mb-na| = 1 $. Hơn nữa $ mb-na = 1 $ nên $ |n(b-a)| = |(m-n)b-1| < |(m-n)n| $ ( do $ b<n $), suy ra $ |b-a|\le |m-n|-1 $. Nhận xét 2 được chứng minh.

Từ $ 2 $ nhận xét trên với điều kiện vị trí ban đầu của con cào cào là $ (1,1) $ ta có
1. Các cặp thỏa mãn là $(m,n) $ nguyên tố cùng nhau
2. Con cào cào có thể nhảy đến sau không quá $|m-n| $ bước.

Traum 09-04-2011 10:17 PM

Bài 3: Max = $\sqrt{2(n-1)(n-2)} $.

Chứng minh: $n(n-1) = x_1^2 + ...+x_n^2 \ge x_1^2+x_2^2 + \frac{n-2}(x_3+...+x_n)^2 = x_1^2 + x_2^2 + \frac{1}{n-2}(x_1+x_2)^2 \ge \frac{1}{2}(x_1+x_2)^2 + \frac{1}{n-2}(x_1+x_2)^2 = \frac{n}{2(n-2)}(x_1+x_2)^2. $ Do đó $(x_1+x_2)^2\le 2(n-1)(n-2) $.
Dấu đẳng thức xảy ra chẳng hạn $x_1 = x_2 = \sqrt{\frac{(n-1)(n-2)}{2}}, $ $x_3 = x_3 =...=x_n = -\sqrt{\frac{2(n-1)}{n-2}} $


Phần min có lẽ là $= 2 $ nếu $n\ge 4 $ ví dụ với $x_1=...=x_{n-1} = 1, x_{n}=-n+1 $.
Với $n = 3 $ thì min = $1 $ ví dụ $x_1 = 2,x_2=x_3 = -1 $

hien123 10-04-2011 12:00 AM

1 Attachment(s)
Trích:

Nguyên văn bởi huynhcongbang (Post 89715)
Mình ủng hộ bài 2 trước:
Trước hết, ta chứng minh bổ đề sau:
* Cho tam giác ABC ngoại tiếp (I) có tiếp điểm của (I) lên AB, AC lần lượt là E, F. Đường thẳng qua B, song song với AC cắt EF tại K; CK cắt AB tại G. Chứng minh rằng tam giác AGI vuông tại I.

Chứng minh:
Do BK // AC nên tam giác BKF cân tại B, suy ra: $BK=BF = p-b $.

[Only registered and activated users can see links. Click Here To Register...]

Theo định lí Thales thì:
$\frac{BG}{AG}=\frac{BK}{AC}=\frac{p-b}{b} \Rightarrow \frac{AB}{AG} = \frac{p}{b}\Rightarrow AG = \frac{bc}{p} $
Mà $AF=p-a $ nên $\frac{AF}{AG}=\frac{p(p-a)}{bc} $.
Ta cũng có: $AI = \frac{AF}{\sin \frac{A}{2}}, AH = AF. \sin \frac{A}{2} $.
Do đó: $\frac{AH}{AI}=\sin^2 \frac{A}{2} = \frac{1-\cos A}{2} = \frac{1-\dfrac{b^2+c^2-a^2}{2bc}}{2} = \frac{p(p-a)}{bc} $
Suy ra: $\frac{AF}{AG} =\frac{AH}{AI} $.
Tức là AGI vuông tại I.
Bổ đề được chứng minh.

Trở lại bài toán:

[Only registered and activated users can see links. Click Here To Register...]

1/ Gọi M, N lần lượt là giao điểm của QE với AB và PF với AC.
Theo bổ đề trên, ta thấy rằng tam giác OMA và ONA lần lượt vuông tại O nên các điểm M, N cố định.
2/ Đặt $AB=AC=a, BP=x, CQ=y $. Chu vi của tam giác APQ là $2(a+x+y) $.
Theo bổ đề trên, ta tính được:
$PM =AP - \frac{2AP.PQ}{AP+AQ+PQ}=\frac{(a+x)x}{a+x+y} $ và
$QN =AQ - \frac{2AQ.PQ}{AP+AQ+PQ}=\frac{(a+y)y}{a+x+y} $.
Ta sẽ chứng minh rằng $\frac{xy(a+x)(a+y)}{(a+x+y)^2} $ không đổi.
Thật vậy:
Diện tích của tam giác APQ cùng bằng:
$R(AP+AQ+PQ) = \sin \widehat{BAC}.AP.AQ \Leftrightarrow \frac{(a+x)(a+y)}{a+x+y}=\frac{R}{ \sin \widehat{BAC}} $.
Tức là tỉ số: $\frac{(a+x)(a+y)}{a+x+y} = k $ không đổi, với $k=\frac{R}{ \sin \widehat{BAC}} $
Từ $(a+x)(a+y)=k(a+x+y) \Leftrightarrow a(a+x+y)+xy = k(a+x+y) \Leftrightarrow a+ \frac{xy}{a+x+y} = k $, suy ra tỉ số $\frac{xy}{a+x+y} $ cũng không đổi.
Ta có đpcm.

Một cách khác:

kien10a1 10-04-2011 12:15 AM

Nếu vậy thì cho em hỏi kết luận của phần a bài 1 là gì ạ?

Traum 10-04-2011 12:26 AM

Trích:

Nguyên văn bởi kien10a1 (Post 89796)
Nếu vậy thì cho em hỏi kết luận của phần a bài 1 là gì ạ?

Quên mất, đáp số là tất cả các cặp $(m,n) $ nguyên tố cùng nhau.

hoanghai_vovn 10-04-2011 02:39 AM

Trích:

Nguyên văn bởi hien123 (Post 89793)
Một cách khác:

Bạn có thể giải thích cho mình tại sao ở câu a, hai tam giác PMO và ONQ đồng dạng với nhau không? Mình nghĩ nếu chứng minh được điều này thì có ngay M, O, N thẳng hàng và MN // BC rồi còn gì?

hien123 10-04-2011 08:41 AM

Trích:

Nguyên văn bởi hoanghai_vovn (Post 89802)
Bạn có thể giải thích cho mình tại sao ở câu a, hai tam giác PMO và ONQ đồng dạng với nhau không? Mình nghĩ nếu chứng minh được điều này thì có ngay M, O, N thẳng hàng và MN // BC rồi còn gì?

Mình sửa lại rồi đó.

can_hang2008 10-04-2011 08:44 AM

Trích:

Nguyên văn bởi nvthanh1994 (Post 89698)
Nhìn qua thấy chắc bài 3 khó nhất.

Bài 3 không khó như em nghĩ đâu. Thật ra dạng bài này đã từng xuất hiện rồi, chỉ khác một chút thôi.

Nhưng bài này anh thấy rất hay vì nó "kill" được thói quen "học dạng" của nhiều bạn.

nbkschool 10-04-2011 10:14 AM

Đề câu 1 có thiếu không nhỉ?Các điểm nó nhảy qua buộc phải có tọa độ nguyên hay thế nào?Nếu không cần thì chỉ cần 3 bước là max thôi.:-??


Múi giờ GMT. Hiện tại là 11:56 AM.

Powered by: vBulletin Copyright ©2000-2024, Jelsoft Enterprises Ltd.

[page compression: 30.17 k/31.59 k (4.49%)]